LSAT and Law School Admissions Forum

Get expert LSAT preparation and law school admissions advice from PowerScore Test Preparation.

User avatar
 Dave Killoran
PowerScore Staff
  • PowerScore Staff
  • Posts: 5853
  • Joined: Mar 25, 2011
|
#26563
Complete Question Explanation
(The complete setup for this game can be found here: lsat/viewtopic.php?t=11096)

The correct answer choice is (A)

Answer choice (A) is the correct answer.

Answer choice (B) is incorrect because it violates the first rule.

Answer choice (C) is incorrect because O must be performed first or fifth.

Answer choice (D) is incorrect because there is not at least one composition between S and O, a violation of the last rule.

Answer choice (E) is incorrect because R and F are not separated by at least two compositions, a violation of the second rule.

Get the most out of your LSAT Prep Plus subscription.

Analyze and track your performance with our Testing and Analytics Package.